Difference between revisions of "2021 Fall AMC 10B Problems/Problem 12"

(Solution 1)
(Tag: New redirect)
Line 1: Line 1:
==Problem 12==
+
#REDIRECT [[2021_Fall_AMC_12A_Problems/Problem_1]]
Which of the following conditions is sufficient to guarantee that integers <math>x</math>, <math>y</math>, and <math>z</math> satisfy the equation
 
<cmath>x(x-y)+y(y-z)+z(z-x) = 1?</cmath><math>\textbf{(A)}\: x>y</math> and <math>y=z</math>
 
<math>\textbf{(B)}\: x=y-1</math> and <math>y=z-1</math>
 
<math>\textbf{(C)} \: x=z+1</math> and <math>y=x+1</math>
 
<math>\textbf{(D)} \: x=z</math> and <math>y-1=x</math>
 
<math>\textbf{(E)} \: x+y+z=1</math>
 
 
 
==Solution 1==
 
Plugging in every choice, we see that choice <math>\textbf{(D)}</math> works.
 
 
 
 
 
We have <math>y=x+1, z=x</math>, so
 
<cmath>x(x-y)+y(y-z)+z(z-x)=x(x-(x+1))+(x+1)((x+1)-x)+x(x-x)=x(-1)+(x+1)(1)=1.</cmath>
 
Our answer is <math>\textbf{(D)}</math>.
 
 
 
~kingofpineapplz
 
 
 
==See Also==
 
{{AMC10 box|year=2021 Fall|ab=B|num-a=13|num-b=11}}
 
{{MAA Notice}}
 

Revision as of 23:30, 23 November 2021